Calculus 3: Finding Current Rate of Change in Electric Circuits Using Chain Rule

Click For Summary
In the context of a simple electric circuit governed by Ohm's law (V = IR), the discussion focuses on finding the rate of change of current (dI/dt) as the battery voltage decreases and resistance increases. The voltage is decreasing at 0.03 volts per second, and resistance is increasing at 0.02 ohms per second. It is emphasized that the differentiation should be performed with respect to time (t) rather than current (I). The correct approach involves applying implicit differentiation and the product rule to determine dI/dt. Understanding these principles is crucial for accurately calculating the current's rate of change in this scenario.
krtica
Messages
50
Reaction score
0
In a simple electric circuit, Ohm's law states that V = IR, where V is the voltage in volts, I is the current in amperes, and R is the resistance in ohms. Assume that, as the battery wears out, the voltage decreases at 0.03 volts per second and, as the resistor heats up, the resistance is increasing at 0.02 ohms per second. When the resistance is 400 ohms and the current is 0.04 amperes, at what rate is the current changing?

Would I differentiate implicitly with respect to the current, I?
 
Physics news on Phys.org
start out by differentiating both sides with respect to time, try using the product rule on the right hand side.
 
To expand on what tt2348 said, you would NOT differentiate with respect to I. The question is "at what rate is the current changing?" This implies that you should take the derivative of I with respect to something (an excellent candidate would be t), not differentiate with respect to I.

IOW, they're asking for dI/dt, the time rate of change of current.
 
Question: A clock's minute hand has length 4 and its hour hand has length 3. What is the distance between the tips at the moment when it is increasing most rapidly?(Putnam Exam Question) Answer: Making assumption that both the hands moves at constant angular velocities, the answer is ## \sqrt{7} .## But don't you think this assumption is somewhat doubtful and wrong?

Similar threads

  • · Replies 20 ·
Replies
20
Views
2K
  • · Replies 1 ·
Replies
1
Views
4K
Replies
15
Views
2K
  • · Replies 7 ·
Replies
7
Views
2K
  • · Replies 25 ·
Replies
25
Views
5K
  • · Replies 57 ·
2
Replies
57
Views
13K
  • · Replies 15 ·
Replies
15
Views
3K
  • · Replies 2 ·
Replies
2
Views
2K
Replies
3
Views
2K
  • · Replies 3 ·
Replies
3
Views
2K